-3
$\begingroup$

Possible Duplicate:
Limit of argz and r

$z_{n} = r_{n}e^{i\theta_{n}}$ and $z = re^{i\theta}$. If $z_{n} \rightarrow z$ then $r_{n} \rightarrow r$ and $\theta_{n} \rightarrow \theta$. $z_{n},z \in G = C - \{z:z \leq 0\}$ and $ - \pi < \theta, \theta_{n} < \pi$.

  • 0
    It is not clear what you are asking. If you browse a bit the site, you will immediately notice that people asking questions usually write up a little bit, explaining what the problem is.2012-09-18
  • 0
    There doesn't seem to be a question here. Are you asking how to prove that if $z_n\to z$ and have the properties stated then $r_n\to r$ and $\theta_n\to\theta$? Also, it would be helpful for you to state your background so that people know at what level they should answer the question.2012-09-18
  • 0
    Also, you just asked this question [here](http://math.stackexchange.com/questions/198525/limit-of-argz-and-r). Please refrain from doing this.2012-09-18

2 Answers 2